Mathcenter Forum

Mathcenter Forum (https://www.mathcenter.net/forum/index.php)
-   อสมการ (https://www.mathcenter.net/forum/forumdisplay.php?f=18)
-   -   อสมการ (https://www.mathcenter.net/forum/showthread.php?t=786)

aaaa 25 กุมภาพันธ์ 2005 09:17

อสมการ
 
หายไปนานกับโจทย์อสมการ กลับมาอีกครั้งพร้อมด้วยโจทย์อสมการหลายๆแบบ

1) (BMO'99) กำหนดให้ \( p,q,r\geq0\) โดย \( p+q+r=1 \) จงพิสูจน์ว่า
\[
ึ7(pq+qr+rp)\leq2+9pqr
\]
2) (Belarus'99) กำหนดให้ \( a,b,c\geq0\) และ \( a^2+b^2+c^2=3\) จงพิสูจน์ว่า
\[
\frac{1}{1+ab}+\frac{1}{1+bc}+\frac{1}{1+ca}\geq\frac{3}{2}
\]
3) (Carlson's inequality) กำหนดให้ \(a,b,c\geq0 \) จงแสดงว่า
\[
\sqrt[3]{\frac{(a+b)(b+c)(c+a)}{8}}\geq\sqrt{\frac{ab+bc+ca}{3}}
\]
4) (French'05) กำหนดให้ \( x,y,z\geq0 \) โดย \( x^2+y^2+z^2=25 \) จงหาค่าต่ำสุดของ
\[
\frac{xy}{z}+\frac{yz}{x}+\frac{zx}{y}
\]
5) (IMO Shortlist'96) กำหนดให้ \( a,b,c>0 \) โดย \( abc=1 \) จงพิสูจน์ว่า
\[
\frac{ab}{a^5+b^5+ab}+\frac{bc}{b^5+c^5+bc}+\frac{ca}{c^5+a^5+ca}\leq1
\]

nooonuii 25 กุมภาพันธ์ 2005 09:44

ขอข้อสองก่อนละกันครับ

\[ \frac{1}{1+ab} + \frac{1}{1+bc}+\frac{1}{1+ca}\geq \frac{9}{(1+ab)+(1+bc)+(1+ca)}\geq \frac{9}{3+a^2+b^2+c^2} = \frac{3}{2} \]

nooonuii 25 กุมภาพันธ์ 2005 10:05

ข้อ 5 ครับ

\[ a^2b^2(a+b) \leq \frac{a^4+b^4}{2} (a+b) \leq a^5 + b^5 \]

\[ a^2b^2(a+b+c) = a^2b^2(a+b) + a^2b^2c =a^2b^2(a+b) + ab \leq a^5+b^5 +ab \]

\[ \frac{ab}{a^5+b^5+ab} \leq \frac{1}{ab(a+b+c)} \]

\[ \frac{ab}{a^5+b^5+ab} + \frac{bc}{b^5+c^5+bc} + \frac{ca}{c^5+a^5+ca} \leq \frac{1}{a+b+c} (\frac{1}{ab}+\frac{1}{bc}+\frac{1}{ca}) = \frac{1}{abc} = 1 \]

nooonuii 25 กุมภาพันธ์ 2005 10:18

ข้อนี้เป็นโจทย์อสมการล่าสุดที่ผมสร้างไว้ครับ ว่าจะเอามาเล่นช่วงปีใหม่แต่ไม่ได้เล่น :D

6. ให้ a,b,c >0 โดยที่ \( a^2 + b^2 + c^2 = 12 \) จงหาค่าสูงสุดของ

\[ \sqrt{a+\sqrt{b+c}} + \sqrt{b+\sqrt{c+a}} + \sqrt{c+\sqrt{a+b}} \]

aaaa 25 กุมภาพันธ์ 2005 10:36

ใช้ อสมการ AM-GM ได้ว่า
\[
\sqrt{a+\sqrt{b+c}}=\frac{1}{2}2\cdot\sqrt{a+\sqrt{b+c}}\leq\frac{1}{4} \left(2^2+a+\sqrt{b+c}\right)\leq\frac{1}{2}\left(4+a+\frac{1}{4}(2^2+b+c)\right)
\]
และเนื่องจาก
\[
a+b+c\leq\sqrt{3(a^2+b^2+c^2)}=6
\]
ดังนั้นจบ

gools 25 กุมภาพันธ์ 2005 22:34

ข้อ 1 ครับ
เนื่องจาก \(p(1-3r)(q-2)\)\((\frac{p-3r+q-1}{3})^{3}=(\frac{1-4r-1}{3})^{3}=\frac{-64r^{3}}{27}\)\(0\)
\p(1-3r)(q-2)+q(1-3p)(r-2)+r(1-3q)(p-2)0
(p-3rp)(q-2)+(q-3pq)(r-2)+(r-3qr)(p-2)0
p(q-2)+q(r-2)+r(p-2)3pq(r-2)+3qr(p-2)+3rp(q-2)
pq+qr+rp-2(p+q+r)3pqr+3pqr+3pqr-6pq-6qr-6rp
เนื่องจาก p+q+r=1 ดังนั้น
7(pq+qr+rp)2+9pqr

nooonuii 26 กุมภาพันธ์ 2005 05:33

โจทย์ข้อ 6 ที่คุณ aaaa เฉลยไว้ ผมลองดูแล้วรู้สึกว่าจะยังไม่ sharp นะครับ
ไม่รู้ว่าผมคิดผิดตรงไหนรึปล่าวเหอเหอ มันลายตาครับ คุณ aaaa ช่วยตรวจทานอีกทีครับ :D

nooonuii 26 กุมภาพันธ์ 2005 05:59

ข้อสามครับ

\( (a+b)(b+c)(c+a) = (a+b+c)(ab+bc+ca) - abc \)

\(\large{ = (a+b+c)(ab+bc+ca) - \sqrt[3]{(ab)(bc)(ca)} \sqrt[3]{abc} } \)

\( \geq (a+b+c)(ab+bc+ca) - (\frac{ab+bc+ca}{3})( \frac{a+b+c}{3}) \)

\( = \frac{8}{9} (a+b+c)(ab+bc+ca) \)

\( \geq 8 (\frac{ab+bc+ca}{3})^{3/2} \)

อสมการสุดท้ายมาจาก \( 3(ab+bc+ca) \leq (a+b+c)^2 \)

aaaa 26 กุมภาพันธ์ 2005 06:00

ขออภัยครับคุณ nooonuii เมื่อวานรีบมากไปหน่อย คือที่มาของวิธีทำคือ ผมมองว่าอสมการจะเท่ากับก็ต่อเมื่อ \( a=b=c=2\)
ซึ่งในกรณีนี้จะได้ว่า
\[
\sqrt{a+\sqrt{b+c}}=\sqrt{b+c}=2
\]
เทอมอื่นก็เช่นเดียวกัน ดังนั้นผมเลยเอาเทอม 2 คูณเข้าและหารออกและใช้ AM-GM จะได้
\[
\sqrt{a+\sqrt{b+c}}=\frac{1}{2}\left(2\cdot\sqrt{a+\sqrt{b+c}}\right)\leq\frac{1}{4}\left(2^2+(a+\sqrt{b+c})\right)
\]
ทำทำนองเดียวกับ เทอม \( \sqrt{b+c} \) จะได้ว่า
\[
\sqrt{b+c}\leq\frac{1}{4}\left(2^2+b+c\right)
\]
ดังนั้นเราจะได้ว่า
\[
\sum_{\text{sym}}\sqrt{a+\sqrt{b+c}}\leq\sum_{\text{sym}}\frac{1}{4}\left(4+a+\frac{1}{4}(4+b+c)\right)=\frac{15}{4}+\frac{3}{8} (a+b+c)
\]
ก็เหลือแสดงว่า
\[
a+b+c\leq6
\]
ซึ่งจะได้ตามมาว่า ค่ามากสุดเท่ากับ 6

nooonuii 26 กุมภาพันธ์ 2005 06:11

เข้าใจแล้วครับ ปรากฎว่า อสมการสุดท้ายของบรรทัดแรก ที่คุณ aaaa เฉลยไว้ตอนแรกมันเป็น 1/2 นี่เองครับ จริงๆมันต้อง 1/4 ผมก็เอาตรงนี้มาบวกกันมันเลยได้เป็น 12 แทน

วิธีคิดของผม ใช้เครื่องมือกันคนละแบบครับ

\[ \sqrt{a+\sqrt{b+c}} + \sqrt{b+\sqrt{c+a}} + \sqrt{c+\sqrt{a+b}} \leq \sqrt{3(a+b+c+\sqrt{a+b} + \sqrt{b+c} + \sqrt{c+a})} \leq \sqrt{3(a+b+c + \sqrt{6(a+b+c)})}\]

แล้วก็ใช้ a+b+c 6 จะได้ค่าสูงสุดเป็น 6 ครับ

nooonuii 26 กุมภาพันธ์ 2005 06:27

7. ให้ a,b,c เป็นจำนวนจริง จงพิสูจน์ว่า
\[ \large { \sqrt{a^2 + (1-b)^2} + \sqrt{b^2 + (1-c)^2} + \sqrt{c^2 + (1-a)^2} \geq \frac{3\sqrt{2}}{2} } \]

aaaa 26 กุมภาพันธ์ 2005 06:36

ขออนุญาติให้ข้อสังเกตุนะครับ

ข้อ 5) ในวิธีทำของคุณ nooonuii สวยงามมากทีเดียวครับ แต่ตอนแรกผมก็มองอสมการที่สองของบรรทัดแรกอยู่นาน เพิ่งจะมองออกว่าใช้
rearrangement ineq นั่นเองครับ วิธีผมคิดเป็นดังนี้ครับ (อยากให้เสนอแนวคิดกันหลายๆแบบครับ จะได้เป็นการแลกเปลี่ยนกัน)
พิจารณาเทอม
\[
a^5+b^5=(a+b)\left[a^4-a^3b+a^2b^2-ab^3+b^4\right]=(a+b)\left[(a-b)^2(a^2+ab+b^2)+(ab)^2\right]\geq(a+b)(ab)^2
\]
ดังนั้น
\[
\frac{ab}{a^5+b^5+ab}\leq\frac{ab}{(a+b)(ab)^2+ab}=\frac{1}{(a+b)/c+1}=\frac{c}{a+b+c}
\]
ทำนองเดียวกันสำหรับเทอมอื่นๆ ดังนั้นก็จบ

ข้อ 6) อืมผมว่าผมทำแบบ basic มากไปเลยออกมายืดยาว แบบของคุณ nooonuii สั้นกว่าเยอะเลยครับ
แต่แนวคิดก็เหมือนกัน :)

ข้อ 1) ของ gools ตรงการใช้อสมการ AM-GM ในบรรทัดแรกนี่ ผมว่าน่าจะผิดครับ เพราะอสมการ AM-GM ต้องใช้เมื่อทุกเทอมมากกว่าหรือเท่ากับศูนย์

nooonuii 26 กุมภาพันธ์ 2005 08:46

ข้อ 4 ครับ

จาก 3(ab+bc+ca) (a+b+c)2 จะได้

\[ \large{ 3[(xy)^2(yz)^2+(yz)^2(zx)^2+(zx)^2(xy)^2] \leq [(xy)^2+(yz)^2+(zx)^2]^2 } \]

\[ \large{ 3(xyz)^2[x^2+y^2+z^2] \leq [(xy)^2+(yz)^2+(zx)^2]^2} \]

\[ \large{ \frac{xy}{z} + \frac{yz}{x} + \frac{zx}{y} \geq 5\sqrt{3} } \]

nooonuii 26 กุมภาพันธ์ 2005 08:54

ข้อ 1 ของน้อง gool นี่ตอนแรกผมคิดว่าน่าจะ modify ได้ แต่ปรากฎว่าทำไม่ได้ครับ เฮ้อคงต้องรอคุณ aaaa มาเฉลยแล้วล่ะครับ เพราะข้อนี้ผมคิดมาอย่างน้อยสามรอบแล้ว โจทย์ BMO นี่ยากจริงๆครับ

nooonuii 26 กุมภาพันธ์ 2005 09:11

โจทย์ของเวียดนาม คู่แข่งที่นำหน้าเราอยู่ประมาณห้าพันลี้ครับ

8. Vietnam'96 ให้ a,b,c,d 0 โดยที่ 2(ab+bc+cd+da+ac+bd) + abc + bcd + cda + dab = 16 จงแสดงว่า
\[ 3(a+b+c+d) \geq 2(ab+bc+cd+da+ac+bd) \]

aaaa 26 กุมภาพันธ์ 2005 09:24

ว้าวคุณ nooonuii นี่เข้าขั้นเซียนด้าน อสมการแล้วนะครับเนี่ย ทำเกือบหมดแล้ว
สำหรับข้อ 1) ต้องพิจารณาผลคูณ
\[
(7-9p)(7-9q)(7-9r)=343-441(p+q+r)+567(pq+qr+rp)-729pqr
\]
และพิสูจน์ว่า
\[
(7-9p)(7-9q)(7-9r)\leq 64
\]
โดยแยกคิดเป็นสองกรณีคือ (1) เมื่อทุกเทอม \( 7-9p,7-9q,7-9r\geq0\) ซึ่งอสมการเป็นผลจาก AM-GM และ (2) อีกกรณีคือมีเทอมเพียงเทอมเดียวที่น้อยกว่าศูนย์ ซึ่งอสมการเป็นจริงเพราะผลคูนน้อยกว่าหรือเท่ากับศูนย์

nooonuii 26 กุมภาพันธ์ 2005 10:04

โห สุดยอดจริงๆครับ เป็นโจทย์ที่น่ากลัวมากๆ :eek: :eek:

nooonuii 26 กุมภาพันธ์ 2005 22:37

คิดและสร้างโจทย์อสมการคืองานอดิเรกของผมตอนนี้ครับ :D

9. ให้ a,b,c > 0 โดยที่
\( \large{ a+b^3+c^5 = 3 } \)
จงหาค่าสูงสุดของ a + 3b + 5c

10. กำหนดให้ a,b,c,d>0 โดยที่
\( \large{\frac{1}{ab} +\frac{1}{bc}+\frac{1}{cd}+\frac{1}{da} = 1} \)
จงพิสูจน์ว่า
\[ \large{ \frac{a^n+b^n+c^n+d^n}{4} \geq 2^n } \]
ทุกจำนวนนับ n

nooonuii 26 กุมภาพันธ์ 2005 22:44

11. ให้ a,b,c>0 โดยที่ a + b + c = 1 จงพิสูจน์ว่า
\[ \large{ \frac{a^4}{b^3+c^3} + \frac{b^4}{c^3+a^3} + \frac{c^4}{a^3+b^3} \geq \frac{1}{2} } \]

gon 26 กุมภาพันธ์ 2005 23:27

ช่วงนี้ไม่ค่อยจะมีเวลาแต่ก็อยากร่วมสนุกครับ. ผมของลองข้อ 1. BMO 1999 อีกรอบล่ะกัน มาดูวิธีผมบ้าง
เพื่อความคุ้นเคยส่วนตัวขอเปลี่ยนโจทย์เป็น

กำหนดให้ \(a, b, c \geq 0 ,\, a + b + c = 1\) จงพิสูจน์ว่า \(7(ab + bc + ca) \leq 9abc + 2\)

ถ้า \(a + b + c = 1\) แล้วจะได้ว่า
\[a^2 + b^2 + c^2 = -2q + 1, \, a^3 + b^3 + c^3 = -3q + 3r + 1\]
เมื่อ \(q = ab + bc + ca, \, r = abc\)

โดยอสมการ Chebyshev จะได้ว่า

\(a^3 + b^3 + c^3 \geq \frac{1}{3}(a + b + c)(a^2 + b^2 + c^2)\)
\(-3q + 3r + 1 \geq \frac{1}{3}(-2q + 1) \Rightarrow 9r + 2 \geq 7q\)
นั่นคือ \(9abc + 2 \geq 7(ab + bc + ca)\) :D

ทำนองเดียวกันจะได้ว่า
\(3(ab + bc + ca)^2 + 6abc + 1 \geq 5(ab + bc + ca) \)
\(2(ab + bc + ca)^2 + 12abc + 5 \geq 8(ab + bc + ca) \)
แต่ดูแล้วไม่งดงามเอาซะเลย

nooonuii 26 กุมภาพันธ์ 2005 23:49

โอ เซียนตัวจริงมาแล้วครับ กำลังรออยู่เลย เป็นวิธีคิดที่สวยมากครับ

gools 27 กุมภาพันธ์ 2005 01:37

ขอบคุณที่เตือนครับคุณ aaaa :p
ข้อ 7 ครับ
\(\begin{array}{rcl}\text{เนื่องจาก }\quad(a+b-1)^{2} & \geq & 0 \\
a^{2}+b^{2}+1-2a-2b+2ab & \geq & 0 \\
2a^{2}+2-4b+2b^{2} & \geq & a^{2}+b^{2}+1+2a-2b-2ab \\
2(a^{2}+(1-b)^{2}) & \geq & (a-b+1)^{2} \\
\sqrt{a^{2}+(1-b)^{2}} & \geq & \frac{(a-b+1)\sqrt{2}}{2} \\
\end{array}
\)
\(
\therefore \qquad \sqrt{a^{2}+(1-b)^{2}}+\sqrt{b^{2}+(1-c)^{2}}+\sqrt{c^{2}+(1-a)^{2}} \geq \frac{(a-b+1)\sqrt{2}}{2}+\frac{(b-c+1)\sqrt{2}}{2}+\frac{(c-a+1)\sqrt{2}}{2} = \frac{3\sqrt{2}}{2}
\)

nooonuii 27 กุมภาพันธ์ 2005 01:59

ข้อ 7 ของน้อง Gool เป็นวิธีคิดที่ดีมากครับ ใช้เฉพาะความรู้พื้นฐานเท่านั้น

nooonuii 27 กุมภาพันธ์ 2005 02:18

ข้อ 7 วิธีคิดของผมครับ

ให้
\( \large{ x= a + (1-b)i } \)
\( \large{ y= b + (1-c)i } \)
\( \large{ z= c + (1-a)i } \)
โดย Triangle inequality ของจำนวนเชิงซ้อนจะได้ว่า
\( \large{ LHS = |x|+|y|+|z| } \)
\( \large{ \geq |x+y+z| } \)
\( \large{ = \sqrt{(a+b+c)^2 + (3-(a+b+c))^2} } \)
\( \large{ \geq \frac{|a+b+c|+|3-(a+b+c)|}{\sqrt{2}} } \)
\( \large{ \geq \frac{|(a+b+c)+(3-(a+b+c))|}{\sqrt{2}} = \frac{3\sqrt{2}}{2} } \)

อสมการในบรรทัดที่สี่มาจาก \( \large{ m+n \leq \sqrt{2(m^2 + n^2)} } \)

aaaa 27 กุมภาพันธ์ 2005 04:49

ข้อ 9) ได้ว่า
\[
9=a+(1+1+b^3)+(1+1+1+1+c^5)\geq a+3\sqrt[3]{b^3}+5\sqrt{c^5}=a+3b+5c
\]
โดยเท่ากับก็ต่อเมื่อ \( a=b=c=1 \)

aaaa 27 กุมภาพันธ์ 2005 04:58

ข้อ 10) ใช้ AM-GM กับเงื่อนไขโจทย์ได้ว่า \( abcd\geq16 \)
และใช้อสมการ AM-GM อีกรอบได้
\[
\frac{a^n+b^n+c^n+d^n}{4}\geq\sqrt[4]{(abcd)^n}\geq2^n
\]

nooonuii 27 กุมภาพันธ์ 2005 08:24

12. ให้ \( x\in (0,\pi/2) \)
จงพิสูจน์ว่า \( \large{ \sin{2x} \geq (\tan{x})^{\cos{2x}} } \)
13. ให้ a,b,c >0 จงพิสูจน์ว่า
\[ \large{ \sqrt[3]{a^3+b^3+c^3} < \sqrt{a^2+b^2+c^2} } \]
14. ให้ a,b,c>0 โดยที่ a+b+c = 1 จงพิสูจน์ว่า
\[ \large{ \frac{2}{3} \leq \frac{\ln{(a^5+b^5+c^5)}}{\ln{(a^7+b^7+c^7)}} \leq \frac{5}{7} } \]

gools 28 กุมภาพันธ์ 2005 15:55

ข้อ 13 ครับ
\(\begin{array}{rcl}\text{เนื่องจาก } a^{2}+b^{2}+c^{2} & > & a^{2},b^{2}\text{ และ }c^{2} \\
\text{ดังนั้น }\sqrt{a^{2}+b^{2}+c^{2}} & > & a,b\text{ และ }c \\
\text{จะได้ว่า }(\sqrt{a^{2}+b^{2}+c^{2}})^{3} & = & (a^{2}+b^{2}+c^{2})\sqrt{a^{2}+b^{2}+c^{2}} \\
& = & a^{2}\sqrt{a^{2}+b^{2}+c^{2}}+b^{2}\sqrt{a^{2}+b^{2}+c^{2}}+c^{2}\sqrt{a^{2}+b^{2}+c^{2}} \\
& > & a^{2}\times a+b^{2}\times b+c^{2}\times c \\
& = & a^{3}+b^{3}+c^{3} \\
\text{ดังนั้น }\qquad \sqrt{a^{2}+b^{2}+c^{2}} & > & \sqrt[3]{a^{3}+b^{3}+c^{3}}
\end{array}\)

gools 28 กุมภาพันธ์ 2005 20:21

ข้อ 11 ครับ
ให้ \(\quad a\geq b\geq c\quad \) โดยไม่เสียนัยทั่วไป เราจะได้ว่า\(\quad a^{4}\geq b^{4}\geq c^{4}\quad\)และ\(\quad\frac{1}{b^{3}+c^{3}}\geq\frac{1}{c^{3}+a^{3}}\geq\frac{1}{a^{3}+b^{3}}\)
โดยอสมการ chebychev จะได้ว่า
\[
\begin{array}{rcl} \frac{a^{4}}{b^{3}+c^{3}}+\frac{b^{4}}{c^{3}+a^{3}}+\frac{c^{4}}{a^{3}+b^{3}} & \geq & \frac{1}{3}(a^{4}+b^{4}+c^{4})(\frac{1}{b^{3}+c^{3}}+\frac{1}{c^{3}+a^{3}}+\frac{1}{a^{3}+b^{3}}) \\
& \geq & \frac{1}{3}(a^{4}+b^{4}+c^{4})(\frac{9}{2(a^{3}+b^{3}+c^{3})}) \\
\text{และเนื่องจาก }\qquad a^{4}+b^{4}+c^{4} & \geq & \frac{1}{3}(a+b+c)(a^{3}+b^{3}+c^{3}) \\
\therefore\qquad \frac{1}{3}(a^{4}+b^{4}+c^{4})(\frac{9}{2(a^{3}+b^{3}+c^{3})}) & \geq & (\frac{1}{3})(\frac{1}{3})(a+b+c)(a^{3}+b^{3}+c^{3})(\frac{9}{2(a^{3}+b^{3}+c^{3})}) \\
& = & \frac{1}{2}
\end{array}
\]

gools 28 กุมภาพันธ์ 2005 21:44

ข้อ 15.
\(\text{ ให้ }a,b,c \geq 0 \text{ จงพิสูจน์ว่า }
\)
\[\frac{a}{\sqrt{a^{2}+8bc}}+\frac{b}{\sqrt{b^{2}+8ca}}+\frac{c}{\sqrt{c^{2}+8ab}} \geq 1\]

gon 01 มีนาคม 2005 20:29

เดี๋ยวแก้ไขก่อนเบลออย่างแรง.

gools 02 มีนาคม 2005 11:07

ขออภัยครับ พอดีรีบไปหน่อย :p
แก้ไขแล้วนะครับ

gools 02 มีนาคม 2005 20:57

คือผมเจอคำถามคล้ายกับข้อ 11. ในartofproblemsolving อยู่ในรูป \(\sum \frac{x^{4}}{x^{3}+y^{3}}\) แล้วก็มีคนที่ชื่อ rodja เสนอให้พิสูจน์ในกรณีทั่วไป โดยเปลี่ยนคำถามใหม่เป็น
ให้ \(x,y,z>0\) และ \(a \geq b > 0\) จงพิสูจน์ว่า \[\frac{x^a}{y^b+z^b}+\frac{y^a}{z^b+x^b}+\frac{z^a}{x^b+y^b} \geq \frac{x^{a-b}+y^{a-b}+z^{a-b}}{2}\] แล้วก็มีคนตอบเข้ามาว่า
It is not symmetric at all.Thus, what rodja posted is NOT a generalization.In fact, rodja's problem is much simpler.
หมายความว่ายังไงครับ(ไม่ค่อยเชี่ยวอังกฤษ :D )

gon 04 มีนาคม 2005 10:36

ช่วงนี้ยังไม่มี้เวลามาคิดต่อปัญหาที่เหลืออยู่บ้างเลยครับ. วุ่นจริง ๆ :rolleyes: เดี๋ยวอีก 3 - 4 วัน น่าจะคงว่างบ้างแล้วจะมาร่วมแก้ต่อ.

nooonuii 05 มีนาคม 2005 09:24

โจทย์ข้อ 15 ของน้อง gool นี่ยากมากทีเดียวครับ เป็นโจทย์ IMO'2001 ผมก็คิดไม่ออกเหมือนกัน แต่มีเฉลยที่สสวท.ทำไว้ครับ เป็นวิธีพิสูจน์ที่สวยงามหมดจดจริงๆครับ

จะพิสูจน์ว่า
\[ \large{ \frac{a}{\sqrt{a^2+8bc}} \geq \frac{a^{4/3}}{a^{4/3} + b^{4/3} + c^{4/3}} } \]

ซึ่งสมมูลกับ
\[ (b^{4/3} + c^{4/3})(a^{4/3} + a^{4/3} + b^{4/3} + c^{4/3}) \geq 8a^{2/3}bc \]

ซึ่งพิสูจน์โดยใช้ AM-GM กับทั้งสองเทอมก็จะได้อสมการที่ต้องการครับ

เข้าไปดูในเวบอื่นรู้สึกว่าจะมีวิธีที่ใช้ Cauchy's inequality ด้วยครับ แต่ผมว่ายากกว่าวิธีนี้

สำหรับข้ออื่นยังมีคนแอบคิดอยู่รึปล่าวครับเนี่ย :D

aaaa 05 มีนาคม 2005 19:49

ข้อ 15 ยากมากทีเดียวครับ แต่มีวิธีคิดแบบอื่นเหมือนกัน ผมยังบอกไม่ได้ตอนนี้ เพราะเร็วๆนี้จะมีข้อสอบแข่งขัน (เป็นโจทย์ inequality) ที่ใช้แนวคิดใหม่นี้ :p

gon 07 มีนาคม 2005 15:03

ข้อ 14 นี่มีอะไรแปลก ๆ หรือเปล่าครับ. :rolleyes:

โดยอสมการ Jensen : สำหรับทุกจำนวนจริงบวก \(a_1, a_2, \cdots , a_n , p, q\) และ \(p < q, n \geq 2 \Rightarrow \sum_{i=1}^{n}(a_i^p)^{\frac{1}{p}} > \sum_{i=1}^{n}(a_i^q)^{\frac{1}{q}} \)

\(\Rightarrow (a^5 + b^5 + c^5)^{\frac{1}{5}} > (a^7 + b^7 + c^7)^{\frac{1}{7}} \Rightarrow (a^5 + b^5 + c^5)^7 > (a^7 + b^7 + c^7)^5 \)
\(\Rightarrow 7\ln(a^5 + b^5 + c^5) > 5\ln(a^7 + b^7 + c^7)\Rightarrow \quad \displaystyle{ \frac{\ln(a^5 + b^5 + c^5)}{\ln(a^7 + b^7 + c^7)} > \frac{5}{7} }\)

warut 07 มีนาคม 2005 15:24

ผมเดาว่า ln(a7 + b7 + c7) มันน้อยกว่า 0 น่ะครับ เวลาย้ายข้างเครื่องหมายเลยต้องกลับด้านด้วย

nooonuii 09 มีนาคม 2005 02:58

อ่า...ใช่แล้วครับ พอใส่ ln แล้วมันจะติดลบเลยต้องกลับเครื่องหมาย ส่วนอีกข้างใบ้ให้ว่าใช้ Holder's Inequality ครับ สำหรับข้ออื่นๆ ยังมีใครคิดอยู่รึปล่าวครับ :)

gools 11 มีนาคม 2005 18:16

ข้อ 8 จะพิสูจน์ในกรณีทั่วไปนะครับ
ให้จำนวนนับ \(n \geq 3\) และ \(a_1,...,a_n\) เป็นจำนวนที่ไม่ใช่จำนวนจริงลบ ให้
\[(n-2) \sum_{i<j} a_i a_j + \sum_{i<j<k} a_i a_j a_k = \frac{2n(n-1)(n-2)}{3}\]
จงพิสูจน์ว่า
\[\sum_{1 \leq i \leq n} a_i \geq \frac{2}{n-1} \sum_{i<j} a_i a_j\]
จะพิสูจน์ในกรณี \(n=3\)
ให้ \(ab+bc+ca+abc=4\)
ต้องพิสูจน์ว่า \(a+b+c \geq ab+bc+ca\)
\[
\begin{array}{rcl}
ab+bc+ca+abc&=&4\qquad \\
\text{ให้ }a \leq b \leq c \text{ โดยไม่เสียนัยทั่วไป จะได้ว่า }3a^{2}+a^{3} &\leq& 4 \\
\therefore\qquad a &\leq& 1 \\
\text{ทำให้มี }c \geq 1 \text{ และ }b&=&\frac{4-ac}{a+b+ac} \\
\text{ แทนค่า }b\text{ ใน }a+b+c-ab-bc-ca\text{ จะได้ว่า} \\
&&a+b+c-ab-bc-ca \\
&=&a(a+c+ac)+4-ac+c{a+c+ac}-a(4-ac)-c(4-ac)-ac(a+c+ac) \\
&=&(a+c-2)^{2}+ac(c-1)(1-a) \geq 0 \qquad \therefore\text{ เป็นจริง} \\
\end{array}
\]
จะพิสูจน์ในกรณี \(n=4\)
ให้ \(f(x)=x^{4}-Ax^{3}+Bx^{2}-Cx+D=0\)
ให้ \(a,b,c,d\) เป็นคำตอบของสมการ จะได้ว่า
\[
A=a+b+c+d,B=ab+ac+ad+bc+bd+cd,C=abc+bcd+cda+dab
\]
diff ทั้ง 2 ข้างของสมการ จะได้ว่า \(4x^{3}-3Ax^{2}+2Bx-C=0\)
ให้ \(\alpha,\beta,\gamma\) เป็นคำตอบของสมการ
จะได้ว่า \(\alpha+\beta+\gamma=\frac{3A}{4},\alpha\beta+\beta\gamma+\gamma\alpha=\frac{B}{2},\alpha\beta\gamma=\frac{C}{4}\)
\(\alpha\beta+\beta\gamma+\gamma\alpha+\alpha\beta\gamma=\frac{2B+C}{4}=4\qquad\qquad\text{ เนื่องจาก }(2B+C=16)\)
เนื่องจาก \(\alpha\beta+\beta\gamma+\gamma\alpha+\alpha\beta\gamma=4\)
ดังนั้น \(\alpha+\beta+\gamma=\frac{3A}{4} \geq \alpha\beta+\beta\gamma+\gamma\alpha=\frac{B}{2}\)
จะได้ว่า \(A \geq \frac{2}{3}B\)
เนื่องจาก \(A=a+b+c+d \text{ และ } B=ab+ac+ad+bc+bd+cd\)
ดังนั้นอสมการเป็นจริงใน \(n=4\)

เมื่อพิสูจน์กรณี \(n=5 \text{ ก็สร้าง } g(x)\) ขึ้นมาแล้วพิสูจน์โดยใช้วิธีเดียวกับกรณี \(n=4\) ครับ ทำอย่างนี้ไปเรื่อยๆ โดยใช้วิธีเดิมก็จะได้ว่าอสมการข้างต้นที่เราต้องการพิสูจน์เป็นจริง


เวลาที่แสดงทั้งหมด เป็นเวลาที่ประเทศไทย (GMT +7) ขณะนี้เป็นเวลา 11:21

Powered by vBulletin® Copyright ©2000 - 2024, Jelsoft Enterprises Ltd.
Modified by Jetsada Karnpracha